Checking Satisfiability of Compound Statement

  • Thread starter Thread starter VinnyCee
  • Start date Start date
Click For Summary
The compound statement is satisfiable, as demonstrated by setting p = TRUE and q = FALSE, which satisfies all conditions due to the OR logical connectives. The discussion emphasizes using logical equivalences such as De Morgan's and Absorption laws to simplify the expression rather than relying solely on truth tables. Participants highlight the importance of systematic approaches over random guessing to find satisfying assignments. The second part of the statement is also addressed, suggesting similar methods for determining satisfiability. Overall, the conversation focuses on logical reasoning techniques for evaluating compound statements.
VinnyCee
Messages
486
Reaction score
0

Homework Statement



Is this compound statement satisfiable?

(p\,\vee\,q\,\vee\,\neg\,r)\,\wedge\,(p\,\vee\,\neg\,q\,\vee\,\neg\,s)\,\wedge\,(p\,\vee\,\neg\,r\,\vee\,\neg\,s)\,\wedge\,(\neg\,p\,\vee\,\neg\,q\,\vee\,\neg\,s)\,\wedge\,(p\,\vee\,q\,\vee\,\neg\,s)

Homework Equations



I guess you are supposed to use the following instead of truth tables somehow:

Logical equivalences - Domination, Idempotent, Double negation, Commutative, De Morgan's, Absorption, Negation, Associate, Distributive.

The Attempt at a Solution



I "converted" the first term in the expression:

(p\,\vee\,q\,\vee\,\neg\,r)\,\equiv\,\left[(\neg\,p\,\longrightarrow\,q)\,\vee\,\neg\,r\right]

Now what do I do though?
 
Physics news on Phys.org
I figured it out!

It was simple.

The equation is satifiable.

Set p = TRUE and q = FALSE. Since it is all OR logical connectives, it was simple to find these values which make the statement true.
 
I really hope you didn't just try things at random. Just use those laws above. It is quite straight forward.
 


can you "VinnyCee" answer this with some explanations

*Is this compound statement is satisfiable/why?


(b) (¬p ∨ ¬q ∨ r) ∧ (¬p ∨ q ∨ ¬s) ∧ (p ∨ ¬q ∨ ¬s) ∧
(¬p ∨ ¬r∨ ¬s)∧ (p ∨ q ∨ ¬r)(¬p ∨ ¬r∨ s)
 
Question: A clock's minute hand has length 4 and its hour hand has length 3. What is the distance between the tips at the moment when it is increasing most rapidly?(Putnam Exam Question) Answer: Making assumption that both the hands moves at constant angular velocities, the answer is ## \sqrt{7} .## But don't you think this assumption is somewhat doubtful and wrong?

Similar threads

  • · Replies 2 ·
Replies
2
Views
2K
  • · Replies 4 ·
Replies
4
Views
11K
Replies
20
Views
2K
  • · Replies 1 ·
Replies
1
Views
2K
  • · Replies 5 ·
Replies
5
Views
7K
Replies
2
Views
37K
  • · Replies 6 ·
Replies
6
Views
2K
  • · Replies 2 ·
Replies
2
Views
3K
Replies
4
Views
3K
  • · Replies 4 ·
Replies
4
Views
6K